Search found 84 matches


OG 12 DS #166

If n is a positive integer is (1/10)^n < 0.01? (1) n > 2 I can understand why (1) is sufficient (2) (1/10)^(n-1) < 0.1 But for statement 2 If I plug in 1 for n then (1/10) ^0 = (1/1) = 1 which is greater than .1 If I plug in 2 for n then (1/10)^ 1 = (1/10) = .1 which is equal to .1 If I plug in 3 fo...

by sudi760mba

Sat Jan 22, 2011 2:39 pm
Forum: Data Sufficiency
Topic: OG 12 DS #166
Replies: 1
Views: 1136

Kaplan Online Free GMAT Test

I just took the Kaplan Online Free Test and scored a 620(Q 39 V 38) and am wondering how the content and score would to the real GMAT.

Thanks.

by sudi760mba

Sun Jan 02, 2011 10:43 pm
Forum: GMAT Strategy
Topic: Kaplan Online Free GMAT Test
Replies: 1
Views: 1653

gmat prep

Six machines, each working at the same constant rate, together can complete a certain job in 12 days. How many additional machines, each working at the same constant rate, will be needed to complete the job in 8 days?" A) 2 B) 3 C) 4 D) 6 E) 7 1) 6 machines complete a job in 12 days. translates...

by sudi760mba

Sun Jun 13, 2010 3:26 pm
Forum: Problem Solving
Topic: gmat prep
Replies: 1
Views: 1429

gmat prep

Six machines, each working at the same constant rate, together can complete a certain job in 12 days. How many additional machines, each working at the same constant rate, will be needed to complete the job in 8 days?" A) 2 B) 3 C) 4 D) 6 E) 7 1) 6 machines complete a job in 12 days. translates...

by sudi760mba

Sun Jun 13, 2010 3:26 pm
Forum: Problem Solving
Topic: gmat prep
Replies: 1
Views: 1112

A list of measurements in increasing order is 4,5,6,8,10 and x. If the median of these measurements is 6/7 times their arithmetic mean, what is the value of x? A. 16 B. 15 C. 14 D. 13 E. 12 Please explain.... 6/7Average = Median Since there are 6 numbers, the median should be between the 3rd and 4t...

by sudi760mba

Sun Jun 13, 2010 2:50 pm
Forum: Problem Solving
Topic: GMAT PREP QUESTION?
Replies: 2
Views: 6645

IMO C. benefited => concerned...parallel? agree on past event.

by sudi760mba

Wed Jun 09, 2010 6:35 am
Forum: Sentence Correction
Topic: Republicans
Replies: 5
Views: 1105

I picked D at first then I saw the OA as C. I can see why. The fact that the militairy no longer requires immunization shot for it's pilots despite a growing threat appears to be contradictory. That can only mean that the real reason is to support the argument in reducing accidents.[/spoiler]

by sudi760mba

Tue Jun 08, 2010 12:38 am
Forum: Critical Reasoning
Topic: airplane accidents
Replies: 16
Views: 3980

From another post, it appears that As well as heat and light are the source of atomic particles...The correct usage of D follows the idiom not only x but also y.

by sudi760mba

Tue Jun 08, 2010 12:00 am
Forum: Sentence Correction
Topic: The Sun
Replies: 3
Views: 3239

20 + p(20) = 20 + 20p
60 - p(60)= 60 - 60p

20 + 20p = 60-60p
80p = 40
p = 40/80 = 1/2

p% of 70 is 1/(2/100) = 1/200 * 70 = 70/200 = 7/20 = 35%

by sudi760mba

Mon Jun 07, 2010 11:25 pm
Forum: Problem Solving
Topic: simple but confusing
Replies: 4
Views: 1377

I think it is C because from the sequence it shows that A through the process will always lead to D. Hence If D does not exist then A was never there. Because if A were there then it would lead to B, then C and then D. But if there is no D then there was no A. I know it is real confusing but hope I...

by sudi760mba

Sat Mar 20, 2010 3:36 pm
Forum: Critical Reasoning
Topic: If then
Replies: 9
Views: 2816

I'm trying a different approach to solution 2. Please comment as to whether this is accurate; Thanks. 5^(k-1)= 5^k - 500 *5^(k-1) is the same as (5^k/5^1)* so (5^k/5^1) = 5^k - 500 *Next move the 5^K to the left hands side from the right hand side* so (5^k/5^1) - 5^k = -500 * Then multiply each term...

by sudi760mba

Wed Dec 16, 2009 4:44 pm
Forum: Data Sufficiency
Topic: Gmat Plus DS
Replies: 6
Views: 4485

Quant Review #133

This should be fairly easy but the explanation in the back of the Quant Review doesn't make too much sense. Any help is appreciated.

If x not equal to 2, then [3x^2(x-2)-x+2]/(x-2)

A)3x^2-x+2
B)3x^2+1
C)3x^2
D)3x^2-1
E)3x^2-2

by sudi760mba

Sat Oct 03, 2009 12:34 pm
Forum: Problem Solving
Topic: Quant Review #133
Replies: 1
Views: 1059

Re: WORD PROBLEM

I've found that some of these inequality problems often introduce a level of ambiguity making them insufficient. Anytime you can refute the established case, or establish a case that contradicts the earlier case, the answer is going to be insufficient. b = Boys g = Girls b > g? (1) g < 3b b= 5 3b = ...

by sudi760mba

Thu Sep 17, 2009 12:25 am
Forum: Data Sufficiency
Topic: WORD PROBLEM
Replies: 3
Views: 1166

I'm in agreement with C as the alternative displaced the established entity. What is the OA?

by sudi760mba

Thu Sep 17, 2009 12:12 am
Forum: Critical Reasoning
Topic: LSAT parallel reasoning
Replies: 6
Views: 1758
by sudi760mba

Thu Sep 17, 2009 12:08 am
Forum: Critical Reasoning
Topic: Tough CR from GPREP
Replies: 12
Views: 2342